Difference between revisions of "1989 AJHSME Problems/Problem 12"

(New page: ==Problem== <math>\frac{1-\frac{1}{3}}{1-\frac{1}{2}} =</math> <math>\text{(A)}\ \frac{1}{3} \qquad \text{(B)}\ \frac{2}{3} \qquad \text{(C)}\ \frac{3}{4} \qquad \text{(D)}\ \frac{3}{2} ...)
 
 
Line 17: Line 17:
 
{{AJHSME box|year=1989|num-b=11|num-a=13}}
 
{{AJHSME box|year=1989|num-b=11|num-a=13}}
 
[[Category:Introductory Algebra Problems]]
 
[[Category:Introductory Algebra Problems]]
 +
{{MAA Notice}}

Latest revision as of 23:57, 4 July 2013

Problem

$\frac{1-\frac{1}{3}}{1-\frac{1}{2}} =$

$\text{(A)}\ \frac{1}{3} \qquad \text{(B)}\ \frac{2}{3} \qquad \text{(C)}\ \frac{3}{4} \qquad \text{(D)}\ \frac{3}{2} \qquad \text{(E)}\ \frac{4}{3}$

Solution

\begin{align*} \frac{1-\frac{1}{3}}{1-\frac{1}{2}} &= \frac{\frac{2}{3}}{\frac{1}{2}} \\ &= \frac{2}{3} \times 2 \\ &= \frac{4}{3} \rightarrow \boxed{\text{E}} \end{align*}

See Also

1989 AJHSME (ProblemsAnswer KeyResources)
Preceded by
Problem 11
Followed by
Problem 13
1 2 3 4 5 6 7 8 9 10 11 12 13 14 15 16 17 18 19 20 21 22 23 24 25
All AJHSME/AMC 8 Problems and Solutions

The problems on this page are copyrighted by the Mathematical Association of America's American Mathematics Competitions. AMC logo.png